A lighthouse casts a 128-ft shadow. A nearby lamppost that measures 5
feet casts an 8-foot shadow. What is the height of the light house, rounded
to the nearest foot? *

Answers

Answer 1

Answer:

80 ft

Step-by-step explanation:

Let the height of the light house be x feet.

[tex] \therefore \: \frac{128}{8} = \frac{x}{5} \\ \\ x = \frac{128 \times 5}{8} \\ \\ x = \frac{640}{8} \\ \\ x = 80 \: feet[/tex]


Related Questions

What is the answer 1/3 of 12 = 1/4

Answers

Step-by-step explanation:

1/3*12=1/4

1/4=1/4

=1/4÷1/4

=0 answer

HELP!!!!! meeeeeejejsjss

Answers

Answer:

[tex]3600[/tex]

Step-by-step explanation:

One is asked to find the cost of a vacation when given the following information;

Travel: 150

Hotel: 50 per day

Spending money: 250

One is asked to calculate the cost of the vacation for (4) people over the course of (7) days. In order to solve this problem, one must make a few assumptions.

- Each cost is per person, therefore one will have to multiply the cost by the number of people.

- The travel is per every one time, thus one will have to multiply it by (2) to account for the cost to travel back home.

- Everyone stays in their own hotel room, therefore, one must multiply the hotel cost by the number of people

- The spending money is for the entire vacation and not per day.

With these assumptions, one can form the following equaton;

x = number of people = 4

y = number of days = 7

[tex]travel\ cost= (2(x(150))\\\\hotel\ cost= (y(x(50))\\\\spending\ money= (x(250))\\\\total\ cost= (travel\ cost)+(hotel\ cost) + (spending\ money)[/tex]

Substitute,

[tex]total\ cost= (travel\ cost)+(hotel\ cost) + (spending\ money)[/tex]

[tex]total\ cost= 2(4(150))+(7(4(50))+(4(250))[/tex]

Simplify,

[tex]total\ cost= 2(4(150))+(7(4(50))+(4(250))[/tex]

[tex]total\ cost= 2(600)+(7(4(50))+(4(250))\\=2(600)+7(200)+4(250)\\=2(600)+7(200)+1000\\=2(600)+1400+1000\\=1200+1400+1000\\=2600+1000\\=3600[/tex]

Find X in this question

Answers

Answer:

Step-by-step explanation:

∠1 = 38    {Vertically opposite angles}

∠2 = 39 + 36 = 75

Exterior angle equals the sum of opposite interior angles.

x = ∠1 +∠2        

 = 38 + 75

x = 113

For this graph, mark the statements that are true.
A. The domain is the set of all real
numbers.
B. The range is the set of all real
numbers greater than or equal to
zero.
C. The domain is the set of all real
numbers greater than or equal to
zero.
D. The range is the set of all real numbers.

Answers

Correct Answer are A and D

Based on the graph, the following statements are true:

A. The domain is the set of all real numbers.

D. The range is the set of all real numbers greater than or equal to zero.

The graph demonstrates that the function will always produce a real number larger than or equal to zero regardless of the real number used as an input. The set of all real numbers constitutes the domain, whereas the set of all real numbers larger than or equal to zero constitutes the range. The other two claims are false. The range is neither the set of all real numbers, nor is the domain restricted to real numbers larger than or equal to zero.

As a result, the right responses are A and D.

To know more about graph:

https://brainly.com/question/17267403

#SPJ2

I’ve gotten this answer wrong twice already

Answers

Answer:

Step-by-step explanation:

The standard vertex form of a quadratic (which is what your graphs are) is

[tex]y=(x-h)^2+k[/tex] where h and  are the coordinates of the vertex. The vertex is really all we need to fill in this equation. Looking at the blue graph, the vertex is located at (-4, 1), so h = -4 and k = 1. Filling in the equation:

[tex]y=(x-(-4))^2+1[/tex] which simplifies down to

[tex]y=(x+4)^2+1[/tex]. It's that simple.I think that's choice C (I'm not sure; it's very tiny!)

Just need 1 answered

Answers

the answer would be ( -7/2 , 7/2 )

Carla packed this box with one centimeter cubes what is the volume of the box plus is it cubic centimeters or just centimeters or is it square centimeters​ pls help you'll get brainlist

Answers

Cubic centimetres since they are cubes. Did you add any picture of the question? There seems to be none.

the answer is cubic centimeters

The volume of a cone is 329.6 cubic inches, and the height is 5.4 inches. Which of the following is the closest to the radius r of the cone, in inches?

Answers

Answer:

329.6=1/3×16.97r

5.66r=329.6/÷5.66

r=58.23

Solve for x. Round to the nearest tenth, if necessary.

Answers

Answer:

x ≈ 8.8

Step-by-step explanation:

Using the sine ratio in the right triangle

sin20° = [tex]\frac{opposite}{hypotenuse}[/tex] = [tex]\frac{PQ}{OQ}[/tex] = [tex]\frac{3}{x}[/tex] ( multiply both sides by x )

x × sin20° = 3 ( divide both sides by sin20° )

x = [tex]\frac{3}{sin20}[/tex] ≈ 8.8 ( to the nearest tenth )

Estimate the number of square yards of carpeting needed to cover a floor 10'3" by 15'9.

Answers

Answer:

17.9375 square yards

Step-by-step explanation:

Let us have a common unit

What we have here is the case of inches and ft

10 ft 3 inches

1 ft = 12 inches

so 3 inches is 3/12 = 0.25 ft

= 10+0.25 = 10.25 ft

15 ft 9 in

= 15 + 9/12 = 0.75 + 15 = 15.75 ft

So let us convert to yards ;

Mathematically, 3 ft = 1 yard

so 10.25 ft = 10.25/3 = 3.4167 yards

15.75 ft = 15.75/3 =5.25 yards

So the square yards would be the product of this two;

which is;

(10.25/3) * (15.75/3) = 17.9375 square yards

Tom goes to the book fair where paperback books are $3 and hardback books are $5.50. Travis
buys three paperback and four hardback books. How much change will Travis receive from a $50
bill
1.Equation/Expression:
2. Solve:

Answers

Answer:

1:  50 - (3(3) + 5.5(4))=x

2: okay so we know how much books are brought and how much each cost.

In this case paper backs are 3 dollars each and cost 3 dollars each. So 3 (3) is 9 dollars. The next scenario are hardbacks which are 5.50 each and Tom brought 4 of them. So that makes it 5.5(4) which is 22 dollars. So the total would be 9+22 which is 31 dollars. The question asks for change from a 50 dollar bill which you subtract and 50-31=19

The change would be 19 dollars

What is the difference of the fractions? 3StartFraction 13 over 15 EndFraction – 1Four-fifths 2One-fifteenth 2Five-fifteenths 2Seven-tenths 2Nine-tenths

Answers

Answer:

2One-fifteenth

Step-by-step explanation:

[tex]3\frac{13}{15} - 1\frac{4}{5}[/tex]

[tex]2\frac{13 - 12}{15}[/tex]

2 [tex]\frac{1}{15}[/tex]

Find the x and y intercept for the equation:
5 + 4 = 20 (show your work pls) serious answers only

Answers

Answer:

x = (4,0)

y = (0,5)

Step-by-step explanation:

5x + 4y = 20

5(0) + 4y = 20

(4y ÷ 4) = (20 ÷ 4)

y = 5

5x + 4(0) = 20

(5x ÷ 5) = (20 ÷ 5)

x = 4

x = (4,0)

y = (0,5)

A bag contains 13 blue marbles, 12 red marbles, 6 yellow marbles, and 8 green marbles. What is the probability of picking a red marble, putting that one back and then picking another red marble?

Answers

Answer:

(12/39) * (12/39) or 16/169

Step-by-step explanation:

A man runs at the speed of 10 km/hr. How much time will he take to cover 2000 meters?

Answers

200 hours.

It’s 10km for every one hour
Divide 2000 by 10 and you will be left with the number of km for every one hour.

Solve the equation and enter the value of x below. -4(x - 5) = 60​

Answers

Answer:

The value of x is 10.

Step-by-step explanation:

By question,

-4(x - 5) = 60

or,-4x + 20 = 60

or,-4x = 60 -20

or, -4x = 40

or, x =40/-4

Hence,x=10

Answer:

x = 10

Step-by-step explanation:

-4(x - 5 ) = 60

Solve for x.

-4(x - 5 ) = 60

Step 1 :- Distribute -4.

-4 × x - 4 × -5 = 60

-4x + 20 = 60

Step 2 :- Move constant to the right-hand side and change their sign.

-4x = 60 - 20

Step 3 :- Subtract 20 from 60.

-4x = 40

Step 4 :- Divide both side by -4.

[tex] \frac{ - 4x}{ - 4} = \frac{40}{ - 4} \\ [/tex]

Hence , x = 10

identify the maximum and minimum values of the function y=10cosx in the interval [-2pie, 2pie]. Use your understanding of transformations, not your graphing calculator.

Answers

Answer:

3 x + 2 y + z/ x + y + z ,  x = 2 ,  y = 3 ,  z = 1

tan ( x ) ,  x = − π

cot ( 3 x ) ,  x = 2 π /3

Step-by-step explanation:

Greg buys 60 garden plants at a cost price of $2.00 each to sell in his shop. He sells 25 of them at the profit of 75% and 18 of them at the profit of 35%. He sells the rest of the plants for 4/5 of the cost price calculate the profit or loss he makes from selling 60 plants stating if it is a profit or loss

Answers

Answer:

$43.30 profit

Step-by-step explanation:

Total cost of plant:

60*2 = 120

Greg makes total of:

25*(2 + 0.75*2) + 18*(2 + 0.35*2) + (60 - 25 - 18)*2*4/5 = 163.3

Since Greg mare than cost, he has a profit and the amount is:

163.3 - 120 = 43.3

solve x in this equation 2x+8x=10x​

Answers

x= all real numbers.
any value of x makes this equation true

Answer:

All real numbers

Step-by-step explanation:

2x+8x=10x​

10x = 10x

Use the image below and find the missing angles

Answers

Step-by-step explanation:

w = 34° (alternate interior angles)

x = 34° (vertically opposite angles)

y = 101°

z = 79° (corresponding angles)

A photograph measures five inches wide and six inches long. The picture is enlarged to fit on a wall. If the new larger picture is 150 inches wide, how long is it?
a. 180 inches
b. 250 inches
c. 210 inches
d. 200 inches

Answers

Answer:

A. 180

Step-by-step explanation:

5×6= 30

150+30

= 180

Answer:

a

Step-by-step explanation:

x/150=6/5

x=6/5×150=6×30=180 inches.

3
49:51
What is the solution to the system that is created by the equation Y =-X+6 and the graph shown below?
10
8
6
4
-10-8-6-44
2
4 6
8 10 x
4
46
-8
10
O (-8,-4)
O (-4,-2)
(4.2)
(6.3)
Mark this and retum
Save and Exit
Next
Submit

Answers

If you draw the graph of y = -x + 6 on the above graph the solution to system will be the coordinates of the point of intersection.
the equation of the line drawn above is y = 0.5x

so you can also find the solution by solving the system
y = 0.5x
y = -x + 6

0.5x = -x + 6
1.5x = 6
x = 6/1.5 = 4

and y = 0.5*4 = 2

The answer is (4, 2) - the third choice.

does anyone know how to solve this ?

Answers

Answer:

D) 144π sq. units

Step-by-step explanation:

The formula for finding the surface area of a sphere is 4π

Substitute 6 in for r.  4π = 144π

Answer:

144π sq. units

Step-by-step explanation:

The formula to determine the surface area of a sphere is: 4πr^2

But there’s no digit of pi given. Therefore we will use the formula but ignoring pi:

4πr^2

= 4r^2

= 4(6^2)

= 4(36)

= 144

Surface area is measured in square units.

Therefore the answer is 144π. The answer will be the true answer surface area once we know what π to use. For now, π will be just a variable next to 144 and will be later multiplied to it.

I hope you understand this! Hope I helped!

Can you please help with this question. Please please

Answers

Add the three numbers.

The magical thing about triangles is if they have 3 angles that are the same then 100% they will be the same lengths.

Annnnnd if you open the sum of the 3 angles more than 180degrees then it wouldn’t be a triangle anymore, the lines will open up.

Perimeter is just a fancy way of saying what’s the outside length/distance.

52+30+40 = 122
5.2 * 9 = 46.8

hope this help

Please help me out with this one

Answers

Answer:

[tex]72 cm^{2}[/tex]

Step-by-step explanation:

[tex]===========================================[/tex]

Formulas:

Area of a rectangle/square:

[tex]A=lw[/tex]

Area of a triangle:

[tex]A=bh\frac{1}{2}[/tex]

[tex]===========================================[/tex]

Square:

5*5= 25 cm.

Rectangle#1:

4*5= 20 cm.

Rectangle #2:

3*5= 15 cm.

Triangle (2):

[tex]3*4*\frac{1}{2}=6[/tex]

Multiply by 2 because there are 2 triangles

6*2= 12 cm

Total:

Add them all up.

25+20+15+12= 72

if the sum is 4 and one of the integers is 1 what must the other integer be

Answers

Step-by-step explanation:

The answer to the question is 3

Answer:

3

Step-by-step explanation:

1 + X = 4

4 - 1 = X

3 = X

What is the value of x in the equation??????
.
15 points!!
Please hurry :) ​

Answers

Answer:

-8

Step-by-step explanation:

An ellipse has a co-vertex at (–8, 9) and a foci at (4, 4). If the center of the ellipse is located below the given co-vertex, then what is the equation of the ellipse? Write in standard form. Guide question? 1) What are the coordinates of the center of the ellipse? 2) Is the ellipse horizontal or vertical?

Answers

Answer:

Step-by-step explanation:

“the center of the ellipse is located below the given co-vertex”

Co-vertex and center are vertically aligned, so the ellipse is horizontal.

Equation for horizontal ellipse:

(x-h)²/a² + (y-k)²/b² = 1

with

a² ≥ b²

center (h,k)

vertices (h±a, k)

co-vertices (h, k±b)

foci (h±c,k), c² = a² -b²

One co-vertex is (-8,9), so h = -8.

One focus is (4,4), so k = 4.

Center (h,k) = (-8,4)

c = distance between center and focus = |-8 - 4| = 12

b = |9-k| = 5

a² = c² + b² = 169

(x+8)²/169 + (y-4)²/25 = 1

Como Determinar a equação da reta que passa pelos pontos A(-1, -2) e B(5,2)

Answers

Answer:

A equação da reta é dada por: [tex]y = \frac{2}{3}x - \frac{4}{3}[/tex]

Step-by-step explanation:

Equação de uma reta:

A equação de uma reta tem o seguinte formato:

[tex]y = ax + b[/tex]

Em que a é o coeficiente angular e b é o coeficiente linear.

Coeficiente angular:

Com posse de dois pontos, o coeficiente angular é dado pela mudança em y dividida pela mudança em x.

A(-1, -2) e B(5,2)

Mudança em y: 2 - (-2) = 2 + 2 = 4

Mudança em x: 5 - (-1) = 5 + 1 = 6

Coeficiente angular: [tex]m = \frac{4}{6} = \frac{2}{3}[/tex]

Então:

[tex]y = \frac{2}{3}x + b[/tex]

Coeficiente linear:

Substituindo um ponto na equação, encontra-se o coeficiente linear.

B(5,2)

Quando [tex]x = 5, y = 2[/tex]. Então:

[tex]y = \frac{2}{3}x + b[/tex]

[tex]2 = \frac{2}{3}5 + b[/tex]

[tex]b = 2 - \frac{10}{3} = \frac{6}{3} - \frac{10}{3} = -\frac{4}{3}[/tex]

Então:

[tex]y = \frac{2}{3}x - \frac{4}{3}[/tex]

What is the next term in the sequence below?
-324,108,-36,12

Answers

Answer:

-4

Step-by-step explanation:

bc it is

.......................

Other Questions
Fedural Europe summary in 2 sentancesPls hurry What industries grew because of the war effort and what did they have to do to produce weapons 8. Two rabbits are bred together. One is heterozygous for agouti coat(genotype: Cch) and the other is albino. What are the possible phenotypesof their offspring? Construct a Punnett square to support your answer. A webinar is a type of audio-only conference call.TURE OR FALSE Many business leaders today give up having a private office, and instead work at a desk or cubicle in the middle of the work area to be among other employees. What effect do you think this work area placement has on their power Joshua is a citizen of the United States. He practices the religion of Judaism freely. Which amendment to the Constitution gives Joshua this freedom? The Texas Constitution mandates that money spent on debt service cannot exceed more than what percentage of state revenue Helpppp me plzzzzzzxzz the growth of a fungal hyphae tip is? I need help figuring out what the answer is. Which of the following represents the divisor and the dividend for thesynthetic division problem below? The width of this rectangle is measured as 19.4 correct to 1 decimal place. What is the lower bound for the area of the rectangle summary no the (mother's tears) An economy is employing 2 units of capital, 5 units of raw materials, and 8 units of labour to produce its total output of 640 units. Each unit of capital costs $10, each unit of raw materials, $4, and each unit of labour, $3.92) Refer to the above information. The per unit cost of production in this economy is: A) $1.00. B) $.10. C) $.05. D) $.50. 93) Refer to the above information. If the per unit price of raw materials rises from $4 to $8 and all else remains constant, the per unit cost of production will rise by about: A) 50 percent. B) 30 percent. C) 40 percent. D) 100 percent. 94) Refer to the above information. As a result of the change indicated in the previous question, the aggregate: A) supply curve would shift to the right. B) demand curve would shift to the right. C) demand curve would shift to the left. D) supply curve would shift to the left. You would use scenario analysis when:________.a. Testing how an increase in revenue growth affects the share price b. Performing what-if analysis to support business planning c. Assessing which assumption has the biggest impact on the model d. Comparing different business cases about the future with multiple variables changed Suppose that you and a friend are playing cards and you decide to make a friendly wager. The bet is that you will draw two cards without replacement from a standard deck. If both cards are diamonds, your friend will pay you $296. Otherwise, you have to pay your friend $17.What is the expected value of your bet? the good construction slithers 3/9 kilometers in 3/6 hours . wat is it's speed in terms of kilometers per hour ? a cupcake weighs 3 1/2 ounces. how many cupcakes are there in 8 4/8 ounce package? SHOW PROCESS!!!Will mark brainly!!Thank you! f(x)=-x^2-9x find f(-2)